LSAT and Law School Admissions Forum

Get expert LSAT preparation and law school admissions advice from PowerScore Test Preparation.

User avatar
 Dave Killoran
PowerScore Staff
  • PowerScore Staff
  • Posts: 5852
  • Joined: Mar 25, 2011
|
#60238
Complete Question Explanation
(The complete setup for this game can be found here: lsat/viewtopic.php?t=15932)

The correct answer choice is (C)

As with any List question, simply apply the rules to the answer choices. Remember to apply the rules in order of the easiest to “see” within the answers. In this game, that order would be rule #4, rule #3, rule #2, and then rule #1 (rule #2 and rule #3 are equally easy to see, but we chose to apply rule #3 first since that allows rules #2 and #1—both of which involve F— to be applied together).

Answer choice (A): This answer is eliminated by the first rule. In this instance, there are only two meetings with F, instead of three.

Answer choice (B): This answer choice is incorrect because two of the meetings with F are consecutive, a violation of the second rule.

Answer choice (C): This is the correct answer choice.

Answer choice (D): Because T and S are not a block, this answer choice violates the third rule, and is therefore incorrect.

Answer choice (E): Because M is last, this answer choice violates the fourth rule and is incorrect.

Get the most out of your LSAT Prep Plus subscription.

Analyze and track your performance with our Testing and Analytics Package.